In the first paragraph, the author refers to a highly reputed critic's persistence in believing van Meegeren's forger...

mcduffeeee on February 5, 2019

Question #4

Hello, I diagrammed the general principle for question 4 as: ^SI --> DDSP & HDSM However, in the video, it is diagrammed as: ^ SI --> HDSM Why was DDSP (depress domestic steel prices) not included in the diagram? Thanks

Replies
Create a free account to read and take part in forum discussions.

Already have an account? log in

Ravi on February 5, 2019

@mcduffeeee,

Thanks for your question. Which lesson was this example in? The video
your question is linking to is the introductory LSAT Max video, which
does not have any example questions.

If you could let me know which lesson this is from, I'd be happy to
help you understand this question better.

Based on what you wrote, I have a feeling this question is from LSAT
22, Section 2, Question 16, which reads:

"Allowing more steel imports would depress domestic steel prices and
harm domestic steel manufacturers. Since the present government will
not do anything that would harm the domestic steel industry, it will
not lift restrictions on steel imports."

The pattern of reasoning in the argument above is most similar to that
in which one of the following?

(A) says, "Building construction increases only when people are
confident that the economy is doing well. Therefore, since people are
now confident in the economy we can expect building construction to
increase."

(B) says, "Since workers are already guaranteed the right to a safe
and healthful workplace by law, there is no need for the government to
establish further costly health regulations for people who work all
day at computer terminals."

(C) says, "In countries that have deregulated their airline industry,
many airlines have gone bankrupt. Since many companies in other
transportation industries are in weaker economic condition than were
those airlines, deregulating other transportation industries will
probably result in bankruptcies as well."

(D) says, "The chief executive officer of Silicon, Inc., will probably
not accept stock in the company as a bonus next year, since next
year's tax laws will require companies to pay a new tax on stock given
to executives."

(E) says, "The installation of bright floodlights on campus would
render the astronomy department's telescope useless. The astronomy
department will not support any proposal that would render its
telescope useless; it will therefore not support proposals to install
bright floodlights on campus."

Is this same question from the example you're referring to?

You're asking why DDSP was not included in the diagram? It wasn't
included for simplicity, as the basic gist of the first sentence is
that letting more steel imports in would be very bad for the steel
industry.

You could express the second part of the first sentence as DDSP + HDSM
if you'd like, and we can in this example. The key thing to note is
that regardless of how you choose to diagram the first sentence, it's
important to understand the argument structure of the stimulus.

The argument structure is

MSI - >DDSP and HDSM

which is

A - >B

Then we're told that the present government won't do anything to hard
the domestic steel industry. This means that (DDSP and HDSM) is
failed, so the necessary condition is failed

/B

The conclusion is that the government will not lift restrictions on
steel imports, which means that MSI is failed, so the sufficient
condition is failed.

/A

The full structure is

A - >B

/B

Therefore, /A

This is a valid argument structure, and the answer choice we pick must
match this.

(E) says, "The installation of bright floodlights on campus would
render the astronomy department's telescope useless. The astronomy
department will not support any proposal that would render its
telescope useless; it will therefore not support proposals to install
bright floodlights on campus."

IBF - >RSDTU (first sentence)

/RSDTU (first part of the second sentence)

/IBF (second part of the second sentence)

The argument structure in this answer choice is

A - >B

/B

Therefore,

/A

This is identical to the structure found in the stimulus, so it's the
right answer choice. (E) is our pick.

Does this make sense? Let us know if you have any more questions!

mcduffeeee on February 5, 2019

Yes, that was the question I was referring to! Thank you for the detailed breakdown! Makes ALOT more sense now.

Ravi on February 5, 2019

@mcduffeeee Happy that it makes sense now! Let us know if you have more questions—we're here to help!